The Stacks project

Exercise 111.58.3. Let $A \to B$ be a ring map such that

  1. $A$ is local with maximal ideal $\mathfrak m$,

  2. $A \to B$ is a finite1 ring map,

  3. $A \to B$ is injective (we think of $A$ as a subring of $B$).

Show that there is a prime ideal $\mathfrak q \subset B$ with $\mathfrak m = A \cap \mathfrak q$.

[1] Recall that this means $B$ is finite as an $A$-module.

Comments (0)


Post a comment

Your email address will not be published. Required fields are marked.

In your comment you can use Markdown and LaTeX style mathematics (enclose it like $\pi$). A preview option is available if you wish to see how it works out (just click on the eye in the toolbar).

Unfortunately JavaScript is disabled in your browser, so the comment preview function will not work.

All contributions are licensed under the GNU Free Documentation License.




In order to prevent bots from posting comments, we would like you to prove that you are human. You can do this by filling in the name of the current tag in the following input field. As a reminder, this is tag 0D5I. Beware of the difference between the letter 'O' and the digit '0'.